Stay ahead of learning milestones! Enroll in a class over the summer!

G
Topic
First Poster
Last Poster
Find the value
sqing   0
24 minutes ago
Source: 2025 Tsinghua University
Let $A=\lim_{n\to\infty}(\frac{\pi}{4}+\frac{1}{n})^n  . $ Find the value of $[100A] .$
0 replies
sqing
24 minutes ago
0 replies
Algebra polynomial problem
Pi-rate_91   0
36 minutes ago
If $ p(x) $ is polynomial with minimum degree such that $p(x)=\frac{x}{x^2+3x+2}$ for $x=0,1,2,...,10$ , find $p(-1)$
0 replies
Pi-rate_91
36 minutes ago
0 replies
Number theory
XAN4   0
38 minutes ago
Source: own
Prove that there exists infinitely many positive integers $x,y,z$ such that $x,y,z\ne1$ and $x^x\cdot y^y=z^z$.
0 replies
1 viewing
XAN4
38 minutes ago
0 replies
Hard geo from UKR TST
mshtand1   7
N 40 minutes ago by bin_sherlo
Source: Ukraine IMO 2023 TST P9
Let $ABC$ be a triangle and let its incircle, centred at $I$, touches the side $BC$ at $D$. A line through $A$ intersects the lines $BC$, $BI$ and $CI$ at $X$, $Y$ and $Z$, respectively. The circle $(ABC)$ intersects the circles $(AIY)$ and $(AIZ)$ again at $U$ and $V$, respectively. Prove that the points $D$, $U$, $V$ and $X$ are concyclic.
Proposed by Fedir Yudin and Mykhailo Shtandenko
7 replies
mshtand1
May 5, 2023
bin_sherlo
40 minutes ago
Test from Côte d'Ivoire Diophantine equation
MENELAUSS   2
N an hour ago by Solar Plexsus
determine all triplets $(x;y;z)$ of natural numbers such that
$$y  \quad  \text{is prime }$$
$$y \quad \text{and} \quad 3  \quad \text{does not divide} \quad z$$
$$x^3-y^3=z^2$$
2 replies
MENELAUSS
Yesterday at 3:05 PM
Solar Plexsus
an hour ago
Petya and vasya are playing with ones
egxa   1
N an hour ago by oolite
Source: All Russian 2025 11.6
$100$ ones are written in a circle. Petya and Vasya take turns making \( 10^{10} \) moves each. In each move, Petya chooses 9 consecutive numbers and decreases each by $2$. Vasya chooses $10$ consecutive numbers and increases each by $1$. They alternate turns, starting with Petya. Prove that Vasya can act in such a way that after each of his moves, there are always at least five positive numbers, regardless of how Petya plays.
1 reply
egxa
Friday at 5:13 PM
oolite
an hour ago
Guessing Point is Hard
MarkBcc168   31
N an hour ago by wu2481632
Source: IMO Shortlist 2023 G5
Let $ABC$ be an acute-angled triangle with circumcircle $\omega$ and circumcentre $O$. Points $D\neq B$ and $E\neq C$ lie on $\omega$ such that $BD\perp AC$ and $CE\perp AB$. Let $CO$ meet $AB$ at $X$, and $BO$ meet $AC$ at $Y$.

Prove that the circumcircles of triangles $BXD$ and $CYE$ have an intersection lie on line $AO$.

Ivan Chan Kai Chin, Malaysia
31 replies
1 viewing
MarkBcc168
Jul 17, 2024
wu2481632
an hour ago
IMO Shortlist 2014 G4
hajimbrak   27
N an hour ago by jp62
Consider a fixed circle $\Gamma$ with three fixed points $A, B,$ and $C$ on it. Also, let us fix a real number $\lambda \in(0,1)$. For a variable point $P \not\in\{A, B, C\}$ on $\Gamma$, let $M$ be the point on the segment $CP$ such that $CM =\lambda\cdot  CP$ . Let $Q$ be the second point of intersection of the circumcircles of the triangles $AMP$ and $BMC$. Prove that as $P$ varies, the point $Q$ lies on a fixed circle.

Proposed by Jack Edward Smith, UK
27 replies
hajimbrak
Jul 11, 2015
jp62
an hour ago
density over modulo M
SomeGuy3335   1
N an hour ago by RagvaloD
Let $M$ be a positive integer and let $\alpha$ be an irrational number in $(0,1)$. Show that for every integer $0\leq a < M$, there exists a positive integer $n$ such that $M \mid \lfloor{n \alpha}\rfloor-a$.
1 reply
SomeGuy3335
Today at 4:19 AM
RagvaloD
an hour ago
From Recursion to Inequality
mojyla222   1
N an hour ago by RagvaloD
Source: IDMC 2025 P2
$\{a_n\}_{n\geq 1}$ is a sequence of real numbers with $a_1=1,\;a_2 =2$ such that for all $n\geq 1$
$$a_{n+2}=\dfrac{a_{n+1}^{2}}{1+a_{n}}+a_{n+1}.$$Prove that

$$\dfrac{1}{1+a_{1}+a_{2}}+\dfrac{1}{1+a_{2}+a_{3}}+\cdots + \dfrac{1}{1+a_{1403}+a_{1404}}>\dfrac{2^{1403}-1}{2^{1404}}.$$
Proposed by Mojtaba Zare
1 reply
mojyla222
5 hours ago
RagvaloD
an hour ago
Problem3
samithayohan   114
N 5 hours ago by Retemoeg
Source: IMO 2015 problem 3
Let $ABC$ be an acute triangle with $AB > AC$. Let $\Gamma $ be its circumcircle, $H$ its orthocenter, and $F$ the foot of the altitude from $A$. Let $M$ be the midpoint of $BC$. Let $Q$ be the point on $\Gamma$ such that $\angle HQA = 90^{\circ}$ and let $K$ be the point on $\Gamma$ such that $\angle HKQ = 90^{\circ}$. Assume that the points $A$, $B$, $C$, $K$ and $Q$ are all different and lie on $\Gamma$ in this order.

Prove that the circumcircles of triangles $KQH$ and $FKM$ are tangent to each other.

Proposed by Ukraine
114 replies
samithayohan
Jul 10, 2015
Retemoeg
5 hours ago
Problem3
G H J
G H BBookmark kLocked kLocked NReply
Source: IMO 2015 problem 3
The post below has been deleted. Click to close.
This post has been deleted. Click here to see post.
samithayohan
41 posts
#1 • 17 Y
Y by sydneymark, Davi-8191, Wizard_32, microsoft_office_word, Flow25, itslumi, somlogan, Abidabi, megarnie, HWenslawski, son7, tiendung2006, Toinfinity, Adventure10, Mango247, Rounak_iitr, Funcshun840
Let $ABC$ be an acute triangle with $AB > AC$. Let $\Gamma $ be its circumcircle, $H$ its orthocenter, and $F$ the foot of the altitude from $A$. Let $M$ be the midpoint of $BC$. Let $Q$ be the point on $\Gamma$ such that $\angle HQA = 90^{\circ}$ and let $K$ be the point on $\Gamma$ such that $\angle HKQ = 90^{\circ}$. Assume that the points $A$, $B$, $C$, $K$ and $Q$ are all different and lie on $\Gamma$ in this order.

Prove that the circumcircles of triangles $KQH$ and $FKM$ are tangent to each other.

Proposed by Ukraine
This post has been edited 7 times. Last edited by djmathman, Feb 14, 2020, 4:21 AM
Reason: typo after 4.5 years!
Z K Y
The post below has been deleted. Click to close.
This post has been deleted. Click here to see post.
TelvCohl
2312 posts
#3 • 38 Y
Y by mssmath, anantmudgal09, AdithyaBhaskar, MariusStanean, utkarshgupta, Abubakir, CeuAzul, reveryu, DreamComeTrue, PeppaBear, Tawan, naw.ngs, samoha, Avlon, enhanced, Abbas11235, Limerent, tree_3, OlympusHero, amar_04, myh2910, PIartist, megarnie, SSaad, hakN, Assassino9931, Derpy_Creeper, Jalil_Huseynov, nguyenducmanh2705, Toinfinity, David-Vieta, OronSH, Adventure10, EpicBird08, sabkx, Sedro, Funcshun840, MS_asdfgzxcvb
Let $ \Psi $ be the inversion with center $ H $ that fixed $ \odot (ABC). $ Since $ Q, H, M  $ and the antipode of $ A $ in $ \odot (ABC) $ are collinear, so $ \Psi(Q) $ is the antipode of $ A $ in $ \odot (ABC) $ and note that $ MH=M\Psi(Q) $ we get $ \Psi(M) $ is the reflection of $ H $ in $ Q. $ Clearly, $ \Psi(K) $ is the antipode of $ Q $ in $ \odot (ABC) $ and $ \Psi(F) $ is the reflection of $ H $ in $ A, $ so $ AQ\Psi(Q) \Psi(K) $ is a rectangle and $ A\Psi(K) $ is the perpendicular bisector of $ \Psi(F)\Psi(M), $ hence $ \Psi(Q)\Psi(K) $ is the tangent of $ \odot (\Psi(F)\Psi(K)\Psi(M)) $ $ \equiv $ $ \Psi(\odot (FKM)). $ I.e., $ \odot (HQK) $ is tangent to $ \odot (FKM). $ $ \qquad \blacksquare $
This post has been edited 2 times. Last edited by TelvCohl, Aug 15, 2023, 9:23 AM
Z K Y
The post below has been deleted. Click to close.
This post has been deleted. Click here to see post.
oneplusone
1459 posts
#4 • 50 Y
Y by buratinogigle, rkm0959, proglote, samyfinsher, BBeast, acegikmoqsuwy2000, shinichiman, toto1234567890, CatalinFetoiu, Vietnamisalwaysinmyheart, quangminhltv99, joshy0604, Garfield, utkarshgupta, baopbc, rterte, PRO2000, muffin_cowee, Ankoganit, reveryu, don2001, MathBoy2001, Lam.DL.01, Tawan, Anar24, samoha, qweDota, Adolmephus, thczarif, Supercali, sriraamster, Atpar, A-Thought-Of-God, Abbas11235, OlympusHero, myh2910, Mop2018, hakN, rayfish, Jalil_Huseynov, nguyenducmanh2705, kimyager, Adventure10, EpicBird08, Dansman2838, Gaunter_O_Dim_of_math, Math_legendno12, Sedro, Funcshun840, MS_asdfgzxcvb
Let $AE$ be the diameter of $\Gamma$. Extend $AF$ to meet $\Gamma$ again at $D$. Suppose $\Gamma$ has center $O$.

Step 1: $Q,H,M,E$ are collinear.
Proof: $BECH$ is a parallelogram, so $E,M,H$ are collinear. $AE$ is the diameter, and $\angle AQE=90$, so $Q,H,M,E$ are collinear.

Step 2: circumcircle of $\triangle DHK$ is tangent to $QE$.
Proof: Let $QP$ be the diameter of $\Gamma$, then $P,H,K$ are collinear. Now $\angle HKD = \angle PQD = \angle OQD = 90-\angle QBD$. Note that $\angle QMC=\angle DMC$, so $QBDC$ is harmonic, so $\angle QBD = \angle QMC$. Thus $\angle HKD = 90-\angle QMC=\angle MHD$, proving step 2.

Now let $X$ be the circumcenter of $\triangle KHD$. Then $X$ lies on $BC$ because it is the perpendicular bisector of $HD$, and $XH\perp QE$. So $XK^2 = XH^2 = XF\cdot XM$, which means $XK$ is tangent to circumcircle of $\triangle MFK$. Also since $XK=XH$ and $XH$ is tangent to circumcircle of $\triangle QHK$, $XK$ is also tangent to circumcircle of $\triangle QHK$, and the result follows.
Z K Y
The post below has been deleted. Click to close.
This post has been deleted. Click here to see post.
leminscate
109 posts
#5 • 5 Y
Y by samyfinsher, reveryu, Abdollahpour, e_plus_pi, Adventure10
This can be done with complex numbers with $\Gamma$ as the unit circle. We construct the tangent to $(KQH)$ at $K$ and let it intersect $\Gamma$ at $X\neq K$. Now we can prove that if $A', Q'$ are the points diametrically opposite $A,Q$, respectively, then $A'MHQ, Q'HK$ are collinear.

$\angle XKH = \angle KQH \implies \frac{(x-k)(h-q)}{(h-k)(k-q)} \in \mathbb{R} \implies \frac{(x-k)(q+a)}{(q+k)(q-k)} \in \mathbb{R} \implies \frac{(x-k)(q+a)}{(q+k)(q-k)} = \frac{\frac{k-x}{xk}\frac{q+a}{qa}}{\frac{q+k}{qk}\frac{k-q}{qk}} \implies x=\frac{qk}{a}$.

EDIT: Just realised that since $\angle AQK = \angle QKX$, $QK || AX$ so $x=\frac{qk}{a}$. So the above paragraph is unnecessary.

We want to prove that $\angle FKX=\angle FMK \iff \frac{(f-k)(k-m)}{(x-k)(b-c)} \in \mathbb{R} \iff \frac{(f-k)(k-m)}{(x-k)(b-c)} = \frac{\overline{(f-k)}\overline{(k-m)}}{\frac{k-x}{xk}\frac{c-b}{bc}} \iff (f-k)(k-m) = xkbc\overline{(f-k)}\overline{(k-m)} \iff (f-k)(k-m) = \frac{qk^2bc}{a} \overline{(f-k)}\overline{(k-m)}$.

Now we can calculate $q=\frac{bc(2a+b+c)}{2bc+ab+ac}$ from $aq=-\frac{q-a}{\overline{(q-a)}} = \frac{h+a}{\overline{(h+a)}} = \frac{2a+b+c}{\frac{2}{a}+\frac{1}{b}+\frac{1}{c}}$.
We calculate $k=a\frac{(a+b+c)(3bc+ab+ac)+abc}{(ab+bc+ca)(3a+b+c)+abc}$ from $kq=-\frac{k-q}{\overline{(k-q)}} = \frac{-q-h}{\overline{(-q-h)}} =q\frac{q+h}{1+q\overline{h}}$.
The hard part is calculating $f-k$ and $k-m$.
$k-m = \frac{(2a+b+c)[(b+c)(a^2-bc)-a(b-c)^2]}{2[(ab+bc+ca)(3a+b+c)+abc]}$ and $f-k = \frac{[(b+c)(a^2-bc)+a(b-c)^2](a+b)(a+c)}{2a[(ab+bc+ca)(3a+b+c)+abc]}$. The main step was really finding the factorisation for the numerator of $k-m$, which I did by somewhat guessing the factor $2a+b+c$ by inspection and luck (mostly luck). Then you can guess the form of $f-k$ so that the RTP statement holds. Here's how I sort of guessed that $2a+b+c$ was a factor:

The numerator of $k-m$ is
\begin{align*} & 2a[(a+b+c)(3bc+ab+ac)+abc] - (b+c)[(ab+bc+ca)(3a+b+c)+abc] \\
&= 2a(a+b+c)(ab+bc+ca) + abc(6a+3b+3c) - (ab+bc+ca)(b+c)(3a+b+c) \\
&= 3abc(2a+b+c) - (ab+bc+ca)(b+c)(2a+b+c) - a(ab+bc+ca)(b+c) + 2a(a+b+c)(ab+bc+ca) \\
&= 3abc(2a+b+c) - (ab+bc+ca)(b+c)(2a+b+c) +a(ab+bc+ca)(2a+b+c). \end{align*} The first manipulation was done with the intention of possibly factoring out $ab+bc+ca$, but then the $3abc(2a+b+c)$ appeared and it seemed as if the $3a+b+c$ was "pretty close" to $2a+b+c$ and that possibly $2a+b+c$ would factor out...and it did.

The remainder of the solution is basically conjugating these expressions and substituting everything in.
This post has been edited 2 times. Last edited by leminscate, Jul 11, 2015, 5:03 AM
Reason: Adding some details, fixing typos
Z K Y
The post below has been deleted. Click to close.
This post has been deleted. Click here to see post.
buratinogigle
2344 posts
#9 • 10 Y
Y by THVSH, andria, anantmudgal09, baopbc, don2001, Tawan, PIartist, Kyleray, Adventure10, Mango247
Great idea to draw circumcenter of triangle $KHD$ dear One Plus One. So we must prove that $(KHD)$ is tangent to $QE$. I do a little different from you. We easily seen $\triangle KQH$ and $\triangle KAE$ is right and $\angle KQH=\angle KQE=\angle KAE$, we deduce $\angle KHQ=\angle KEA=\angle KHD$. So we are done.

I propose a general problem and solution

Let $ABC$ be an actue triangle inscribed in circle $(O)$ and $P$ is a point inside it such that $\angle BPC=180^\circ-\angle A$. $BP,CP$ cut $CA,AB$ at $E,F$. Circle $(AEF)$ cuts $(O)$ again at $G$. Circle diameter $PG$ cuts $(O)$ again at $K$. $D$ is projection of $P$ on $BC$ and $M$ is midpoint of $BC$. Prove that the circle $(KPG)$ and circle $(KDM)$ are tangent.

Solution. Let $Q$ are symmetric of $P$ through $D$, easily seen $Q$ lie on $(O)$. Let $GP$ cuts $(O)$ again at $N$. We see $\angle NPC=\angle FPG=\angle FAG=\angle BNP$ so $BN\parallel PC$ similarly, $CN\parallel BP$. Let $AS,NR$ are diameter of $(O)$. We eaily seen $\angle PQN=90^\circ$ so $P,Q,R$ are collinear. So $M$ is midpoint of $PN$. We have $\angle KPG=90^\circ-\angle KGP=90^\circ-\angle KAN=\angle AKN+\angle ANK-90^\circ=\angle NKS+\angle KQA=\angle NAS+\angle KQA=\angle ANR+\angle KQA=\angle AQR+\angle KQA=\angle KQP.$

From this, $GN$ is tangent to $(KPQ)$. Let tangent at $K,P$ of $(KPG)$ intersect at $T$. We have $\angle KTP=180^\circ-2\angle KGP=2\angle KPG=2\angle KQP$ and $TK=TP$, so we have $T$ is circumcenter of triangle $KPQ$, but $BC$ is perpendicular bisector of $PQ$, therefore $T$ lies on $BC$. Now, $TK^2=TP^2=TD.TM$, this means $TK$ is common tangents of $(KGP)$ and circle $(KHM)$ or they are tangent. We are done.
Attachments:
Z K Y
The post below has been deleted. Click to close.
This post has been deleted. Click here to see post.
THVSH
158 posts
#12 • 3 Y
Y by buratinogigle, Adventure10, Mango247
Another generalization:
Let $ABC$ be a triangle with circumcircle $\odot (O)$. A arbitrary circle $\odot (D)$ passing through $B, C$ intersects $CA, AB$ at $E, F$, respectively. $K$ is the orthogonal projection of $D$ on $AP$. Construct the diameter $AP$ of $\odot (AEF)$. $\odot (AEF) \cap \odot (O)=\{A, G\}$. $\odot (GP) \cap \odot (O)=\{G, J\}$. Prove that $\odot (JGP)$ and $\odot (JKD)$ are tangent.
:)
Attachments:
Z K Y
The post below has been deleted. Click to close.
This post has been deleted. Click here to see post.
LeVietAn
375 posts
#13 • 2 Y
Y by Adventure10, Mango247
Another generalization :) :
Let $ABC$ be a triangle with circumcircle $\Gamma$ . A arbitrary circle $(O)$ passing through $B, C$ intersects $CA, AB$ at $D, E$, respectively. Let $H=BD\cap CE$. Let $Q $ be the point on $ \Gamma $ such that $\angle HQA =90^{\circ}$ and let $K $ be the point on $\Gamma $ such that $\angle HKQ=90^{\circ}$.
Let $F$ is the orthogonal projection of $H$ on $BC$, and let $M=KO\cap BC$.
Prove that the circumcircles of triangles $KQH $ and $FKM $ are tangent to each other.
Attachments:
Z K Y
The post below has been deleted. Click to close.
This post has been deleted. Click here to see post.
TelvCohl
2312 posts
#14 • 5 Y
Y by gold46, Tawan, enhanced, Adventure10, Mango247
THVSH wrote:
Another generalization:
Let $ABC$ be a triangle with circumcircle $\odot (O)$. A arbitrary circle $\odot (D)$ passing through $B, C$ intersects $CA, AB$ at $E, F$, respectively. $K$ is the orthogonal projection of $D$ on $AP$. Construct the diameter $AP$ of $\odot (AEF)$. $\odot (AEF) \cap \odot (O)=\{A, G\}$. $\odot (GP) \cap \odot (O)=\{G, J\}$. Prove that $\odot (JGP)$ and $\odot (JKD)$ are tangent.
:)
The solution by oneplusone also works for this generalization :)
I just want to note that step 2 in the solution simplified follows from Reim theorem ($ \because AQEP $ is rectangle $ \Longrightarrow QE \parallel AP $) .
Z K Y
The post below has been deleted. Click to close.
This post has been deleted. Click here to see post.
v_Enhance
6874 posts
#15 • 24 Y
Y by mssmath, rkm0959, silouan, Dukejukem, zschess, aayush-srivastava, huricane, don2001, Tawan, Wizard_32, naw.ngs, MathbugAOPS, Limerent, Purple_Planet, myh2910, SSaad, Assassino9931, nguyenducmanh2705, UI_MathZ_25, Adventure10, Mango247, Tellocan, Rounak_iitr, Sedro
Probably equivalent to the other inversion solutions, but anyways. . .
[asy]size(6cm);
defaultpen(fontsize(9pt));

size(8cm);
pair A = dir(80);
pair B = dir(220);
pair C = dir(-40);

draw(unitcircle, blue);
pair O = circumcenter(A, B, C);
pair H = orthocenter(A, B, C);
pair Q = IP(CP(midpoint(A--H), H), unitcircle);
pair N = midpoint(H--Q);
pair T = midpoint(A--H);
pair N_9 = midpoint(O--H);
pair M = midpoint(B--C);
pair F = foot(A, B, C);

draw(circumcircle(M, F, N), darkcyan);

pair L = M+T-N;
pair K = OP(CP(N, H), unitcircle);

draw(A--B--C--cycle, red);
draw(A--F);
draw(T--M--L--N, heavygreen);
draw(M--Q--A, magenta);
draw(Q--K--L, heavymagenta);

dot("$A$", A, dir(A));
dot("$B$", B, dir(B));
dot("$C$", C, dir(C));
dot("$O$", O, dir(O));
dot("$H$", H, dir(H));
dot("$Q$", Q, dir(Q));
dot("$N$", N, dir(N));
dot("$T$", T, dir(T));
dot("$N_9$", N_9, dir(N_9));
dot("$M$", M, dir(M));
dot("$F$", F, dir(F));
dot("$L$", L, dir(L));
dot("$K$", K, dir(K));

/* Source generated by TSQ */
[/asy]

Let $N$, $T$ be midpoints of $HQ$, $AH$. Let $L$ be on the nine-point circle with $\angle HML = 90^{\circ}$. The negative inversion at $H$ swapping $\Gamma$ and nine-point circle maps $A$ to $F$, $K$ to $L$, $Q$ to $M$. AS $LM \parallel AQ$ we just need to prove $LA = LQ$. But $MT$ is a diameter, hence $LTNM$ is a rectangle, so $LT$ passes through the center of $\Gamma$.
This post has been edited 2 times. Last edited by v_Enhance, Jul 10, 2015, 2:39 PM
Reason: Shrink diagram, edit solution
Z K Y
The post below has been deleted. Click to close.
This post has been deleted. Click here to see post.
ISHO95
221 posts
#16 • 3 Y
Y by Mathuzb, Adventure10, Mango247
Let $R$ be the reflection of $H$ onto $BC$ and $KR$ intersects $BC$ at $T$. We easily prove that $KQMT$ is cyclic. If $KF$ intersects circle $KQH$ at $P$ , then $Q,P,T$ are collinear. => let $l$ be line which tangents circle $KQH$ at $K$. Then it is not hard to see line $l$ also tangents to the circle $KFM$ at $K$. Done!
This post has been edited 2 times. Last edited by ISHO95, Jul 10, 2015, 7:56 PM
Z K Y
The post below has been deleted. Click to close.
This post has been deleted. Click here to see post.
Math-lover123
304 posts
#17 • 2 Y
Y by mssmath, Adventure10
If $O$ is cicumcenter of $ABC$ and $V,S,T$ are midpoints of segments $OH,HK,HQ$ respectively,
then $VS$ and $VT$ are midlines of triangles $HOK$ and $HOQ$ resprectively.
So $S,T$ lie on Euler's circle of $ABC$.
If $t$ is a line through $H$ which is perpendicular to $HM$,
then $t,MF,ST$ are concurrent at radical center $X$ of Euler's circle of $ABC$ and circumcircles of triangles $STH$ and $MFH$.
Triangles $XKS$ and $XHS$ are congruent,
so $XK^2=XH^2=XS\cdot XT=XM\cdot XF$ and $\measuredangle XKS=90$,
which means that the line $XK$ is common tangent of circumcircles of triangles $KFM$ and $HKQ$.
This post has been edited 2 times. Last edited by Math-lover123, Jul 10, 2015, 8:26 PM
Reason: circumcircle
Z K Y
The post below has been deleted. Click to close.
This post has been deleted. Click here to see post.
Kfp
17 posts
#18 • 2 Y
Y by Adventure10, Mango247
I found this problem quite pretty.

First of all, it is well known that $Q$, $H$ and $M$ are collinear. Assuming this, we call $H_{A}$ the second intersection of $AH$ with $\Gamma$, and we perform an inversion of center $H$ and power $HA \cdot HH_{A}$. After very basic angle chasing + application of the fact stated above, we get the problem transformed into the following: let $ABC$ be a triangle and let $H'$ be its incenter. Let $F'$ be its A-excenter (we know that $BF'CH'$ is cyclic for obvious reasons, we call this circle $\omega$), let $Q'$ be the midpoint of the circumcircle arc containing $A$, let $M'$ be the second intersection of $Q'H$ with $\omega$ and let $K'$ be the intersection between $\Gamma^{'}$ and the perpendicular line wrt $HQ'$ passing through $Q'$. We want to show that the circumcircle of $F'M'K'$ is tangent to $Q'K'$ at $K'$.

To do this, we'll show that $K'$ lies on the AXIS of $F'M'$. To show this is enough , once we defined $D$ as the center of $\omega$, to show that $K'D$ is orthogonal to $M'F'$. But $M'F'$ is orthogonal to $M'Q'$ ($HF'$ being a diameter) and $M'Q'$ is orthogonal to $Q'K'$ by definition. But $Q'D'$ is a diameter of $\Gamma^{'}$, hence done!!
This post has been edited 3 times. Last edited by Kfp, Jul 10, 2015, 9:54 PM
Z K Y
The post below has been deleted. Click to close.
This post has been deleted. Click here to see post.
mathuz
1514 posts
#19 • 2 Y
Y by Davrbek, Adventure10
I think it's easy for Problem 3:

We have that two lemmas:

\textbf{Lemma 1.} The points $Q$, $H$ and $M$ are collinear.

\textbf{Lemma 2.} Let $N$ is the midpoint of $QH$ and a point $R$ lies on the line $BC$ such that $RH\parallel AQ$. Then $\angle RNO= 90^{\circ} $, where $O$ is the circumcenter of the triangle $ABC$.

We can use by two lemmas and we easily can see that the circumcircles of the triangles $KHQ$ and $KFM$ are tangent. :)
Z K Y
The post below has been deleted. Click to close.
This post has been deleted. Click here to see post.
proglote
958 posts
#20 • 2 Y
Y by Adventure10, Mango247
Here's a solution with inversion WRT $Q$. First, let $A_0$ denote the antipode of $A$ in $\Gamma$; we know that $Q, H, M, A_0$ are collinear and $M$ is the midpoint of $HA_0$, so that $M'$ becomes the harmonic conjugate of $Q$ WRT $A'_0, H'$ (Note that $A', M', F'$ are collinear since $(QAMF)$ is cyclic):
[asy]
 /* Geogebra to Asymptote conversion, documentation at artofproblemsolving.com/Wiki, go to User:Azjps/geogebra */
import graph; size(15cm); 
real labelscalefactor = 0.5; /* changes label-to-point distance */
pen dps = linewidth(0.7) + fontsize(10); defaultpen(dps); /* default pen style */ 
pen dotstyle = black; /* point style */ 
real xmin = -4.800874036810980, xmax = 34.89250840395043, ymin = -7.788624316942659, ymax = 11.96084728234451;  /* image dimensions */
pen qqttqq = rgb(0.000000000000000,0.2000000000000002,0.000000000000000); pen aqaqaq = rgb(0.6274509803921575,0.6274509803921575,0.6274509803921575); pen qqwuqq = rgb(0.000000000000000,0.3921568627450985,0.000000000000000); 

draw((558.4074274619950,-167.6931808709421)--(558.4848138116372,-168.2773169816810)--(559.0689499223762,-168.1999306320388)--(558.9915635727340,-167.6157945212999)--cycle, qqwuqq); 
draw((11.94814897609412,-2.701786992093843)--(11.83986216803928,-3.280991282635144)--(12.41906645858058,-3.389278090689988)--(12.52735326663543,-2.810073800148687)--cycle, qqwuqq); 
draw((12.24038502123578,-5.722950743110142)--(11.76393233897595,-5.376257096575377)--(11.41723869244118,-5.852709778835205)--(11.89369137470101,-6.199403425369972)--cycle, qqwuqq); 
draw((18.65735096727185,3.095730151442312)--(19.13380364953167,2.749036504907546)--(19.48049729606644,3.225489187167374)--(19.00404461380661,3.572182833702140)--cycle, qqwuqq); 
 /* draw figures */
draw(circle((11.92149259612108,3.614427735914360), 7.082678004468662), linewidth(1.200000000000002) + qqttqq); 
draw((11.89369137470101,-6.199403425369972)--(16.80965888055248,-1.511014497759957), aqaqaq); 
draw((7.004365667404278,-1.483237463161803)--(11.89369137470101,-6.199403425369972), aqaqaq); 
draw((19.00404461380661,3.572182833702140)--(11.89369137470101,-6.199403425369972)); 
draw((11.53043691294743,-3.457446355699668)--(11.89369137470101,-6.199403425369972)); 
draw((5.406156668775245,-1.478709958916395)--(11.89369137470101,-6.199403425369972)); 
draw((5.406156668775245,-1.478709958916395)--(26.02563863289654,-1.537122089126370)); 
draw((19.00404461380661,3.572182833702140)--(26.02563863289654,-1.537122089126370)); 
draw((26.02563863289654,-1.537122089126370)--(8.649924021738130,-3.839056692143185)); 
draw((5.406156668775245,-1.478709958916395)--(14.14050284768400,-3.111664830980842)); 
draw((11.89369137470101,-6.199403425369972)--(12.52735326663543,-2.810073800148687)); 
 /* dots and labels */
dot((19.00404461380661,3.572182833702140),dotstyle); 
label("$Q$", (19.11515275219922,3.738845041291063), NE * labelscalefactor); 
dot((7.004365667404278,-1.483237463161803),dotstyle); 
label("$C'$", (7.143250840395042,-1.205467117180268), NE * labelscalefactor); 
dot((16.80965888055248,-1.511014497759957),dotstyle); 
label("$B'$", (16.42078039617833,-1.122136013385808), NE * labelscalefactor); 
dot((11.89369137470101,-6.199403425369972),dotstyle); 
label("$H'$", (12.25422520645530,-6.177556310249753), NE * labelscalefactor); 
dot((26.02563863289654,-1.537122089126370),dotstyle); 
label("$A'$", (26.14274250553207,-1.372129324769190), NE * labelscalefactor); 
dot((15.30832042658575,-1.506761414320957),dotstyle); 
label("$A'_0$", (14.55971907810204,-1.177690082582115), NE * labelscalefactor); 
dot((14.14050284768400,-3.111664830980842),dotstyle); 
label("$M'$", (14.19861762832604,-3.455406919630706), NE * labelscalefactor); 
dot((11.53043691294743,-3.457446355699668),dotstyle); 
label("$F'$", (10.92092754574392,-3.260967677443630), NE * labelscalefactor); 
dot((5.406156668775245,-1.478709958916395),dotstyle); 
label("$K'$", (5.504405799103985,-1.316575255572882), NE * labelscalefactor); 
dot((8.649924021738130,-3.839056692143185),dotstyle); 
label("$M_0$", (8.143224085928571,-4.233163888379004), SW * 0.1*labelscalefactor); 
dot((12.52735326663543,-2.810073800148687),dotstyle); 
label("$X$", (12.64310369082944,-2.649872916284253), NE * labelscalefactor); 
clip((xmin,ymin)--(xmin,ymax)--(xmax,ymax)--(xmax,ymin)--cycle); 
 /* end of picture */[/asy]
Note that $A'(Q, M'; A'_0, H')$ is a harmonic pencil and $A'Q \parallel H'K' \Longrightarrow A'M'$ cuts $H'K'$ at its midpoint $M_0$. Define $X$ as the foot of the perpendicular from $H'$ to $M'K'$, so that $(H'M'XF')$ all lie on the circle with diameter $H'M'$. Now $\angle K' XF' = \angle F'H'M' = 180 - \angle K'M_0 F'$, i.e. $(K' X F' M_0)$ is cyclic. But note that $M_0 X^2 = M_0 H'^2 = M_0 F' \cdot M_0 M'$, i.e. $M_0 X$ is tangent to $(M' X F')$ and $\angle H' K' F' = \angle M_0 X F' = \angle K' M' F'$, i.e. $H'K'$ is tangent to $(F'K'M')$, i.e. $(KQH)$ is tangent to $(FKM)$ at $K$, as desired.
Z K Y
The post below has been deleted. Click to close.
This post has been deleted. Click here to see post.
Géza Kós
111 posts
#24 • 2 Y
Y by Adventure10, Mango247
I can be found quickly that $Q$ is the intersection of $\Gamma$ and the ray $MH$.

What happens if, by accident, we take the other intersection point? Instead of $Q$ and $K$, we will get the reflections of $H$ about $M$ and $F$, respectively and find that the two circles in the problem statement are trivially tangent to each other.

Now let us draw both sides of the problem in the same picture. We can realize what object can connect the two parts...
Z K Y
G
H
=
a